Which one of the following pairs of employees is such that at least one member of the pair volunteers?

maonuo on November 16, 2019

Question

Why is C wrong?

Replies
Create a free account to read and take part in forum discussions.

Already have an account? log in

Irina on November 16, 2019

@maonuo,

Because it is possible that neither L nor V volunteered.
If L is out, then R, M, T and S are in, and F and V are out, resulting in the following complete list of volunteers:
In: R M T S
Out: L V F

Since the question requires us to find a pair of employees such that at least one of the members must volunteer, and we can come up with a scenario where both L & V are out, we can conclude that (C) is incorrect.

Let me know if you have any other questions.

maonuo on November 16, 2019

Thanks Irinia that makes sense now